subject
Physics, 13.07.2019 05:30 kikipie489

If w is a subspace of r" and if is in both w and w, then must be the zero vector in the orthogonal decomposition theorem, each term in y is itself an orthogonal projection of f onto a subspace of w. lf у z1 +z2, where ź1 is in a subspace w and z2 is in w , then z1 must be the orthogonal projection of y onto w the best approximation to y by elements of a subspace w is given by the vector y-projwy. if an n p matrix u has orthonormal columns, then uutx= for all x in rn

ansver
Answers: 1

Another question on Physics

question
Physics, 21.06.2019 19:30
Which graph best represents the relationship between the electric current and the rate at which a magnet is turning inside an electric generator?
Answers: 3
question
Physics, 22.06.2019 05:50
Sawyer is studying diffraction. he draws a diagram of a plane wave to show how light waves travel. which best describes sawyer’s error? the wave fronts should be perpendicular to the direction in which the waves move. the arrow showing the direction of movement of the waves should be pointing to the left. the arrow showing the direction of movement of the waves should be pointing downward. the wave fronts should be both parallel and perpendicular to the direction in which the waves move.
Answers: 2
question
Physics, 22.06.2019 06:40
Determine the change in width a, height b, thickness t when a plate is subjected to the uniform distributed load and is made of material having modulus of elasticity e=230 gpa and poisson's ratio ν=1/3. given : a=400 mm and b= 300 mm also the uniformly distributed load in downward y direction of plate is 2 mn/m and in the positive x direction is 3 mn/m and t=20 mm
Answers: 1
question
Physics, 22.06.2019 12:10
Aspring has a natural length of 8 m. if a 12-n force is required to keep it stretched to a length of 10 m, how much work w is required to stretch it from 8 m to 16 m? (round your answer to two decimal places.)
Answers: 1
You know the right answer?
If w is a subspace of r" and if is in both w and w, then must be the zero vector in the orthogonal d...
Questions
question
Health, 11.11.2020 16:40
Questions on the website: 13722359